Good CR!!!

This topic has expert replies
User avatar
Legendary Member
Posts: 934
Joined: Tue Nov 09, 2010 5:16 am
Location: AAMCHI MUMBAI LOCAL
Thanked: 63 times
Followed by:14 members

Good CR!!!

by [email protected] » Sun Jan 29, 2012 10:18 pm
Political Analyst: Because our city is a border city, illegal immigration is an important issue in the current race for mayor. Of the two candidates for mayor, one supports a plan that would attempt to deport the city's 9,000 illegal immigrants and the other does not. Surveys consistently show that about 60% of the city's residents are opposed to the plan, while about 35% are in support of the plan. Therefore, the candidate who does not support the plan will win the election for mayor. All of the following statements weaken the analyst's argument, EXCEPT:

"¢ In the city at issue, most voters make their voting decisions based on the candidates' positions on abortion.

"¢ Of the 35% of residents who support the plan, some are willing to consider alternate plans for addressing illegal immigration.

"¢ Many of the residents who oppose the plan are not registered voters.

"¢ The candidate who supports the plan is the incumbent mayor, and has been elected to four consecutive terms despite taking controversial positions on many important issues.

"¢ Just under 30% of the city's residents are illegal immigrants who cannot vote.



The OA is B, but there are a few points to consider in this question. It gives something new that I want to ask.

1 question: wat is the option E really doing? is it weakening the argument or not affecting the argument at all. According to me, it does not affect the argument at all as the probability involved is very high. 30% could be both included in overall 60% as well as 35% of the lot. If it is with the 60% then it negates or weakens the argument and if it is with the 35% of the lot, then it strengthens the argument. So what is the optimal thing that can be done...











Lets say there is a weaken except question like the above question...
This is a new idea that I got, nothing related to this question but just if GMAT plays this trick then what should a test taker do...


Option A weakens the argument.

Option B strengthens the argument.

Option C also weakens the argument.

Option D weakens the argument as well.

Option E does not affect the argument at all.



So what should be done in this scenario.

Should we opt for option E or option B. both are not weakening the argument but wat is the answer choice to be chosen. Could any you explain to me this thing....
IT IS TIME TO BEAT THE GMAT

LEARNING, APPLICATION AND TIMING IS THE FACT OF GMAT AND LIFE AS WELL... KEEP PLAYING!!!

Whenever you feel that my post really helped you to learn something new, please press on the 'THANK' button.

User avatar
Legendary Member
Posts: 641
Joined: Wed Apr 20, 2011 1:15 am
Thanked: 149 times
Followed by:32 members
GMAT Score:760

by avik.ch » Mon Jan 30, 2012 12:22 am
yes !! E do weakens the argument.

60% of the immigrants are opposed to the plan - deporting 9000 illegal immigrants.

So 30 % who cannot vote - belong to this category of people. I dont have much logic to support this but its common sense. Think of something like this - we will never support a bill against corruption and bribery, if we are the ones who benifit the most out of corruption and bribery.

So its now - 30% and 35 % --- this weakens the conclusion.

Hope this helps !!

Legendary Member
Posts: 627
Joined: Thu Jun 23, 2011 9:12 am
Thanked: 4 times
Followed by:1 members

by mankey » Mon Jan 30, 2012 6:01 am
This one's not clear. Please help.

Regards.

User avatar
GMAT Instructor
Posts: 641
Joined: Wed Jul 22, 2009 3:07 pm
Location: Madison, WI
Thanked: 162 times
Followed by:45 members
GMAT Score:760

by Jim@Grockit » Tue Jan 31, 2012 10:15 pm
Even though it's common sense to imagine that the illegal immigrants who cannot vote would oppose the plan, you absolutely cannot assume that the 30% illegal population is entirely contained within the 60%. First, they may be 100% of the 5% who did not answer, putting them at under 25% of the city's residents, leaving the two sides tied at 35%. Second, the question isn't using exact percentages, which also should suggest a trap to you.

User avatar
Legendary Member
Posts: 934
Joined: Tue Nov 09, 2010 5:16 am
Location: AAMCHI MUMBAI LOCAL
Thanked: 63 times
Followed by:14 members

by [email protected] » Tue Jan 31, 2012 11:56 pm
Just under 30% of the city's residents are illegal immigrants who cannot vote.

Yes right, option E also weakens the argument. 30% of the total residents are immigrants and they will be against the proposal. This means that in the 60% opposition, there are a few immigrants that are the residents themselves and so they cannot vote. Hence lets say out of the 60%, 30% is removed then the majority turns to the other side.
Hence weakens the argument...
IT IS TIME TO BEAT THE GMAT

LEARNING, APPLICATION AND TIMING IS THE FACT OF GMAT AND LIFE AS WELL... KEEP PLAYING!!!

Whenever you feel that my post really helped you to learn something new, please press on the 'THANK' button.

User avatar
Legendary Member
Posts: 934
Joined: Tue Nov 09, 2010 5:16 am
Location: AAMCHI MUMBAI LOCAL
Thanked: 63 times
Followed by:14 members

by [email protected] » Wed Feb 01, 2012 12:00 am
Yes Jim is absolutely correct but in GMAT CR the author assumes that the argument is airtight. But in the option E, the argument is not airtight as it can also weaken at the same time strengthen the argument. It is like DS question. So if you feel that the option does not weaken or strengthen an argument completely, then that option gets cancelled off......
IT IS TIME TO BEAT THE GMAT

LEARNING, APPLICATION AND TIMING IS THE FACT OF GMAT AND LIFE AS WELL... KEEP PLAYING!!!

Whenever you feel that my post really helped you to learn something new, please press on the 'THANK' button.

Legendary Member
Posts: 581
Joined: Sun Apr 03, 2011 7:53 am
Thanked: 52 times
Followed by:5 members

by killer1387 » Wed Feb 01, 2012 2:08 am
Hey,
Can u please mention the source of this question.

Thanx.

GMAT/MBA Expert

User avatar
GMAT Instructor
Posts: 3380
Joined: Mon Mar 03, 2008 1:20 am
Thanked: 2256 times
Followed by:1535 members
GMAT Score:800

by lunarpower » Mon Feb 06, 2012 4:41 am
jim --
Jim@Grockit wrote:Even though it's common sense to imagine that the illegal immigrants who cannot vote would oppose the plan, you absolutely cannot assume that the 30% illegal population is entirely contained within the 60%.
almost every strengthen/weaken problem requires common-sense assumptions.

if one could object in this sort of way, then the answer to almost every s/w problem in the whole official canon would be invalidated.
just flipping to a random page in the OG, we have OG12 #65. in that problem, the correct answer relies on the assumption that "the government of kourion just stopped making coins" means "a great big disaster happened". in other words, you have to use common sense to reject other possibilities (e.g., they stopped printing coins because of inflation; they invented magnetic debit cards; they moved to paper money; etc.)

in fact, the whole point of S/W questions is to test test-takers' ability to interweave logical thinking with common sense.

First, they may be 100% of the 5% who did not answer, putting them at under 25% of the city's residents, leaving the two sides tied at 35%.
yeah, but these are "strengthen/weaken" questions, not "confirm/destroy" questions.
taking the split of a:b down from 60:35 to 35:35 obviously weakens the case for "a". it doesn't destroy that case, but, well, it doesn't have to. it's just a significant piece of counter-evidence.
Ron has been teaching various standardized tests for 20 years.

--

Pueden hacerle preguntas a Ron en castellano
Potete chiedere domande a Ron in italiano
On peut poser des questions à Ron en français
Voit esittää kysymyksiä Ron:lle myös suomeksi

--

Quand on se sent bien dans un vêtement, tout peut arriver. Un bon vêtement, c'est un passeport pour le bonheur.

Yves Saint-Laurent

--

Learn more about ron

Master | Next Rank: 500 Posts
Posts: 242
Joined: Fri Aug 01, 2008 8:49 am
Location: Delhi
Thanked: 6 times

by ranjeet75 » Mon Feb 06, 2012 8:19 am
How Option A weakens the argument? Please someone elaborate. I can't understand

GMAT/MBA Expert

User avatar
GMAT Instructor
Posts: 3380
Joined: Mon Mar 03, 2008 1:20 am
Thanked: 2256 times
Followed by:1535 members
GMAT Score:800

by lunarpower » Mon Feb 06, 2012 4:39 pm
ranjeet75 wrote:How Option A weakens the argument? Please someone elaborate. I can't understand
here:
https://www.beatthegmat.com/weaken-cr-t1 ... tml#449499
Ron has been teaching various standardized tests for 20 years.

--

Pueden hacerle preguntas a Ron en castellano
Potete chiedere domande a Ron in italiano
On peut poser des questions à Ron en français
Voit esittää kysymyksiä Ron:lle myös suomeksi

--

Quand on se sent bien dans un vêtement, tout peut arriver. Un bon vêtement, c'est un passeport pour le bonheur.

Yves Saint-Laurent

--

Learn more about ron

User avatar
GMAT Instructor
Posts: 641
Joined: Wed Jul 22, 2009 3:07 pm
Location: Madison, WI
Thanked: 162 times
Followed by:45 members
GMAT Score:760

by Jim@Grockit » Tue Feb 07, 2012 8:35 am
lunarpower wrote:jim --
Jim@Grockit wrote:Even though it's common sense to imagine that the illegal immigrants who cannot vote would oppose the plan, you absolutely cannot assume that the 30% illegal population is entirely contained within the 60%.
almost every strengthen/weaken problem requires common-sense assumptions.

if one could object in this sort of way, then the answer to almost every s/w problem in the whole official canon would be invalidated.
I understand what you are saying, but I would point to the U.S. Republican party as a prime and common-sense example of something that some people vote for/are in favor of, despite it clearly being against their best interest in some respects. There's probably a similar example in a lot of countries. The OG example isn't a great parallel because we do still accept the passage there as true; my point was that here, we have a missing 5% and inexact numbers before we even hit the answer choices.

I think my advice still stands -- be suspicious of questions set up this way -- but I think I focused too much on that. :( I actually wonder whether choice B is a typo for E or whether choice B should read "Of the 60% who oppose the plan", thereby suggesting that they are generally in favor of something being done but against this specific plan.